Nina
Thanks Received: 0
Atticus Finch
Atticus Finch
 
Posts: 103
Joined: October 15th, 2012
 
 
 

Q21 - If Skiff's book is published

by Nina Mon Jul 15, 2013 1:15 pm

I found answer A a little bit incomplete to be a sufficient assumption, because what we need exactly is "skiff's book will be published this year if it is as important and as well written as Skiff's claims", is it correct because it is the best answer left? or should we tolerate the sufficient assumption to be sometimes incomplete?

many thanks!
User avatar
 
ManhattanPrepLSAT1
Thanks Received: 1909
Atticus Finch
Atticus Finch
 
Posts: 2851
Joined: October 07th, 2009
 
This post thanked 3 times.
 
 

Re: Q21 - If Skiff's book is published

by ManhattanPrepLSAT1 Fri Jul 19, 2013 2:55 pm

I understand why it feels incomplete, but trust me it's not.

Suppose I give you the following hypothetical argument:

A --> B
B --> C
---------
D + A --> C

The conclusion contains extraneous information, but it is a valid argument. The combination of D + A will ensure that C is true, because A by itself is all that you need to ensure C.

The same thing happens here.

Published --> N. Urge
N. Urge --> Promote
--------------------------
Imp + Writ --> Promote

If we assume that "important" is enough to ensure that the book will be "published," (Imp --> Published) then the trigger in the conclusion contains enough information to guarantee the outcome. And so answer choice (A) would ensure that the conclusion is valid.

Incorrect Answers
(B) reverses the logical inference that could be derived from the first two premises.
(C) is irrelevant. Professor Nguyen is the connection between being published and being promoted, but he will urge the dean to promote if the book is published.
(D) reverses the direction of the assumption involving both important and well written. While the terms line up better here, it still reverses the logic.
(E) reverses the second premise.

#officialexplanation
 
vania_apple
Thanks Received: 0
Forum Guests
 
Posts: 5
Joined: June 06th, 2013
 
 
 

Re: Q21 - If Skiff's book is published

by vania_apple Tue Sep 10, 2013 3:46 am

So basically would the argument still be valid if we assumed that "well written" is enough ensure the book be published? ie, Skiff's book will be published this year if it is as important as he claims it is.

cheers
 
aamatsui
Thanks Received: 0
Vinny Gambini
Vinny Gambini
 
Posts: 4
Joined: April 03rd, 2013
 
 
 

Re: Q21 - If Skiff's book is published

by aamatsui Mon Sep 23, 2013 7:57 pm

vania_apple Wrote:So basically would the argument still be valid if we assumed that "well written" is enough ensure the book be published? ie, Skiff's book will be published this year if it is as important as he claims it is.

cheers


Yeah I believe so. Here is how I boiled down the problem

Conclusion: If important and well written ---> promote

Premises:

Skiff Published --> Nguyen recommend
Nguyen recommend --> Dean Promotes

A) if important --> published

A) allows us to set off the chain which eventually ends in being promoted by the dean. Well written --> published would have worked the same way... at least I think


For me, the tough part of this question was seeing how Nguyen "urging the dean to promote skiff" was the same as "recommending it".
 
Djjustin818
Thanks Received: 0
Jackie Chiles
Jackie Chiles
 
Posts: 34
Joined: June 15th, 2012
 
 
 

Re: Q21 - If Skiff's book is published

by Djjustin818 Sun Sep 29, 2013 11:51 pm

Not sure if I'm missing something here but I don't see how D reverses the logic.

Published --> N. Urge
N. Urge --> Promote
--------------------------
Imp + Writ --> Promote

That's exactly how I diagramed it. Now I thought D says:

IF NOT important and well written --> Will not be published
Contrapositive: If Published --> Important and Well Written

When you insert that into the argument you get:

Published --> Imp + Writ --> Promote

I've been staring at this problem for 20 minutes trying to figure out what I'm missing but I have no idea how this reverses the logic.. I even re-evaluated my "unless" method. Could you clarify please?
 
wizzard880
Thanks Received: 0
Forum Guests
 
Posts: 7
Joined: March 10th, 2013
 
 
 

Re: Q21 - If Skiff's book is published

by wizzard880 Wed Nov 27, 2013 6:15 pm

Also there's another problem with D. At least that i saw. D says "will not be published" but we need his book to be published this year. So you can eliminate off of that.
 
agutman
Thanks Received: 9
Vinny Gambini
Vinny Gambini
 
Posts: 17
Joined: December 19th, 2012
 
This post thanked 3 times.
 
 

Re: Q21 - If Skiff's book is published

by agutman Tue Dec 10, 2013 4:30 pm

Here's my explanation!

PT69, S4, Q21 (Sufficient Assumption)

This argument starts with a conditional statement, and as we read through the rest of the argument we can count two more! This is a very good indication that formal logic will help us sort out the premises here:

Skiff’s book is published this year --> Prof vows she will urge dean to promote Skiff + Prof will certainly keep her promise --> Prof recommends promotion --> Dean will promote Skiff

If we cut out the middle men, this boils down to:
Skiff’s book is published this year --> Dean will promote Skiff

Thus, if Skiff’s book is as important and as well written as Skiff claims, he will be promoted.

There seems to be a bit of a gap here! It looks like we need something to trigger the book getting published this year. "˜As important and as well written as Skiff claims’ must be the trigger! Since this is a Sufficient Assumption question, there’s a good chance we’ll find exactly what we’re looking for in the answer choices. So, before we go in there, let’s quickly clarify what a good answer would look like: if Skiff’s book is as important and as well written as Skiff claims, it will be published this year.
Ready? Go:

(B) Nothing about important or well written.

(C) we don’t care whether the Prof. thinks the book is well written. Besides, we need it to trigger something and this isn’t even a conditional statement.

(D) Go ahead and translate this into formal logic. You should get the exact opposite direction of what we’re looking for.

(E) Nothing about important or well written.

(A) this doesn’t mention well written. But is it sufficient to trigger what we needed? It sure is! You can think of this answer choice as overfilling our conditional arrow.
 
gaheexlee
Thanks Received: 10
Elle Woods
Elle Woods
 
Posts: 55
Joined: May 27th, 2014
 
 
 

Re: Q21 - If Skiff's book is published

by gaheexlee Wed Sep 24, 2014 4:57 pm

mattsherman Wrote:A --> B
B --> C
---------
D + A --> C

If we assume that "important" is enough to ensure that the book will be "published," (Imp --> Published) then the trigger in the conclusion contains enough information to guarantee the outcome. And so answer choice (A) would ensure that the conclusion is valid.


When you said "if we assume that 'important' is enough to ensure..." you mean we are assuming because the question stem allows us, even requires us, to assume, right?

So if I have a sentence that says "apples and bananas ensure a great dish," I can't correctly assume that apples are sufficient by itself to create a great dish and say "apples ensure a great dish." In this question, answer choice (A) is LETTING us assume that a book's importance can suffice that it will be published even though the stimulus mentioned 2 elements in the sufficient part of the diagram.

Am I correct?
 
dkm8832
Thanks Received: 0
Vinny Gambini
Vinny Gambini
 
Posts: 1
Joined: November 21st, 2014
 
 
 

Re: Q21 - If Skiff's book is published

by dkm8832 Fri Nov 21, 2014 1:38 pm

Hi All, I really love agutman's response to this, so thank you, but I'm left with a big question about your explanation:

(A) this doesn’t mention well written. But is it sufficient to trigger what we needed? It sure is! You can think of this answer choice as overfilling our conditional arrow.


Why is this sufficient to only mention "important" but not mention "well written"? Since what we're looking for is: important and well written --> published, Don't you need to satisfy both conditions in order to trigger the necessary condition?

Can anyone else help explain this?

Thank you!
User avatar
 
maryadkins
Thanks Received: 641
Atticus Finch
Atticus Finch
 
Posts: 1261
Joined: March 23rd, 2011
 
This post thanked 1 time.
 
 

Re: Q21 - If Skiff's book is published

by maryadkins Wed Nov 26, 2014 11:48 am

gaheexlee Wrote:So if I have a sentence that says "apples and bananas ensure a great dish," I can't correctly assume that apples are sufficient by itself to create a great dish and say "apples ensure a great dish." In this question, answer choice (A) is LETTING us assume that a book's importance can suffice that it will be published even though the stimulus mentioned 2 elements in the sufficient part of the diagram.

Am I correct?


Yes, good.

dkm8832 Wrote:Hi All, I really love agutman's response to this, so thank you, but I'm left with a big question about your explanation:

(A) this doesn’t mention well written. But is it sufficient to trigger what we needed? It sure is! You can think of this answer choice as overfilling our conditional arrow.


Why is this sufficient to only mention "important" but not mention "well written"? Since what we're looking for is: important and well written --> published, Don't you need to satisfy both conditions in order to trigger the necessary condition?

Can anyone else help explain this?

Thank you!


Think of this this way. (A) gives you what you need and MORE by leaving out well-written. It basically lowers the standard. It says, actually, being important is enough on its own. You don't even NEED well-written anymore...we're going to bridge the gap for you just based on it being important.
 
Beetlenav
Thanks Received: 0
Vinny Gambini
Vinny Gambini
 
Posts: 1
Joined: December 04th, 2014
 
 
 

Re: Q21 - If Skiff's book is published

by Beetlenav Fri Dec 05, 2014 2:10 am

This question is problematic. If one assumes only that the book is important, then the book could not be well written. In which case the conclusion can not be properly inferred.

The prompt implicitly states that the book must be important AND well written for it to be published this year.

One must therefore assume both in order for the conclusion to be properly inferred.

A is the least wrong answer, but it is none the less insufficient.
 
blahsheep
Thanks Received: 0
Vinny Gambini
Vinny Gambini
 
Posts: 6
Joined: September 02nd, 2013
 
 
 

Re: Q21 - If Skiff's book is published

by blahsheep Sat May 30, 2015 11:48 pm

Beetlenav Wrote:This question is problematic. If one assumes only that the book is important, then the book could not be well written. In which case the conclusion can not be properly inferred.

The prompt implicitly states that the book must be important AND well written for it to be published this year.

One must therefore assume both in order for the conclusion to be properly inferred.

A is the least wrong answer, but it is none the less insufficient.


I don't think the prompt implies the book must be important and well written for it to be published. That would imply that important and well-written are necessary conditions when they're actually sufficient. It says, IF it's important and well written, then S. will be promoted.

(A) basically tells that Important is sufficient to be published. So we have:

If Important, then Published.
If Published, then N. will recommend promotion.
If N. recommends promotion, then dean will promote.

We can truncate that and have If important, then promotion (which is basically the conclusion). Obviously, we actually have if Important AND Well-written, then promotion. If we remember that Important -> Published, then that still makes our answer sufficient (because we're told that Published will be enough for N. to recommend and so on). We could even have Important & NOT well-written and it would STILL be sufficient to ensure a promotion because we know that Important-> Published and so on.

I think the key is to remember that we're dealing with sufficient and not necessary conditions. Well-written and important aren't necessary for a promotion, just like being being published isn't necessary for N. to recommend a promotion. It's entirely possible for S. to get a promotion without being published or without his book being important, or even without N. recommending it to the dean. These are all sufficient conditions.
 
krisk743
Thanks Received: 2
Jackie Chiles
Jackie Chiles
 
Posts: 49
Joined: May 31st, 2017
 
 
 

Re: Q21 - If Skiff's book is published

by krisk743 Wed Jul 26, 2017 1:57 pm

ManhattanPrepLSAT1 Wrote:I understand why it feels incomplete, but trust me it's not.

Suppose I give you the following hypothetical argument:

A --> B
B --> C
---------
D + A --> C

The conclusion contains extraneous information, but it is a valid argument. The combination of D + A will ensure that C is true, because A by itself is all that you need to ensure C.

The same thing happens here.

Published --> N. Urge
N. Urge --> Promote
--------------------------
Imp + Writ --> Promote

If we assume that "important" is enough to ensure that the book will be "published," (Imp --> Published) then the trigger in the conclusion contains enough information to guarantee the outcome. And so answer choice (A) would ensure that the conclusion is valid.

Incorrect Answers
(B) reverses the logical inference that could be derived from the first two premises.
(C) is irrelevant. Professor Nguyen is the connection between being published and being promoted, but he will urge the dean to promote if the book is published.
(D) reverses the direction of the assumption involving both important and well written. While the terms line up better here, it still reverses the logic.
(E) reverses the second premise.










Ok I don't understand in any way how these conditional statements leads to A > B and B > C......

Originally I did:
N. Recommends > Promote
Published > Promote
__________________________
Important and well written > Promote

I don't even get how Published and Recommended are being combined in everyones explanation. I know vows she will urge is equivocated to will certainly keep her promise but they still don't add up.

HOW DO YOU KNOW ONE GOES FIRST!? Does Recommend lead to published or published leads to recommend.....if I knew this earlier I would have gotten to A correctly but left it blank and chose D because of the way I diagrammed it.

"IF Nguyen recommends it, THEN she will keep her promise (to PROMOTE).

Someone please explain